LSAT and Law School Admissions Forum

Get expert LSAT preparation and law school admissions advice from PowerScore Test Preparation.

 Robert Carroll
PowerScore Staff
  • PowerScore Staff
  • Posts: 1787
  • Joined: Dec 06, 2013
|
#63176
Joanna,

The mayor simply states that the effectiveness of the educational programs is debatable. The mayor is not saying they wouldn't work - the mayor is saying there's some question whether they would work. The purpose of this comment by the mayor is to signal that she doesn't intend to judge the effectiveness of those programs because she thinks she can demonstrate the effectiveness of the taxes themselves. The conclusion doesn't involve an opinion on the programs' effectiveness, so an answer that address it will be out of scope.

Robert Carroll
User avatar
 smtq123
  • Posts: 29
  • Joined: May 28, 2021
|
#93587
Hi,

We are told several times (also in this forum) that for Strengthening and Weakening questions, we should not bring any assumption while evaluating the options. So, I am confused in option C, we are assuming that people will go out of the city to buy cheaper cigarettes.

However, I could assume that the sales increase has nothing to do with the city smokers; it is just that people living outside the city are anticipating similar increase in taxes on cigarettes and therefore they are buying them in order to hoard them.

Hence, I get very confused when to make assumption in Strengthening and Weakening questions. If the experts can provide some guidelines on this subject, it will be helpful.

Thanks in advance!
User avatar
 smtq123
  • Posts: 29
  • Joined: May 28, 2021
|
#93589
Hi,

Let me add my other question related to the argument.

In my view, the conclusion is NOT what it is stated in this forum "Taxes would produce the sought after reduction in smoking" rather it is "Taxes THEMSELVES would produce the sought after reduction in smoking" which means "Taxes ALONE would produce the sought after reduction in smoking" .

So, I am inclined towards option D which is indicating that educating people about tobacco use will help reduction in smoking.

Thanks!
 Adam Tyson
PowerScore Staff
  • PowerScore Staff
  • Posts: 5153
  • Joined: Apr 14, 2011
|
#93709
I'll address the second question first here, smtq123, and point out that you are making a classic conditional error in your reasoning there, a Mistaken Reversal. The mayor is arguing that the taxes are SUFFICIENT to bring about a reduction in smoking; answer D addresses whether the taxes are a NECESSARY condition. In saying that taxes alone would bring about the reduction, the mayor is arguing that they COULD do it by themselves even if the education program has no impact. They are not saying that nothing else could have a similar impact, so answer D does not attack that argument.

With regard to answer C, no assumption on our part is required for this answer to raise some doubts about the conclusion. That's the key to a good Weaken answer - it only needs to raise doubts about the conclusion, and so does not need to prove the conclusion is false. Answer C, showing a possible increase in sales nearby, should raise some doubts about the tax actually causing a reduction in smoking, because it COULD indicate that local smokers will just cross city lines to get their smokes (much like when I was a kid in a place where the drinking age was 19 but the town just across the State border set it at 18. Road trip!) Not that it PROVES that local smokers will still smoke just as much - that's too high a standard to require from the answer - but just that it raises that possibility, and by so doing it creates at least some doubt about the mayor's claim.
User avatar
 queenbee
  • Posts: 75
  • Joined: Sep 18, 2022
|
#97482
Hi
I chose B as many others did. Not only did I discount C because it talked about surrounding areas, I felt that B directly talked about the tax impact on sales. I guess my mistake was I didn't identify the conclusion properly. I thought the conclusion was "surveys show that cigarette sales drop when stiff taxes are imposed" but I guess that was a premise? The actual conclusion is "taxes would produce the reduction in smoking (paraphrased)"
Is that right?
thank you!
 Adam Tyson
PowerScore Staff
  • PowerScore Staff
  • Posts: 5153
  • Joined: Apr 14, 2011
|
#97881
Correct! The author is trying to prove that the taxes alone could have caused a drop in smoking, and the evidence is that sales of cigarettes dropped where the tax was imposed. Surrounding areas matter because an increase in sales there could mean that smokers didn't change their smoking habits, just their point of purchase!

Get the most out of your LSAT Prep Plus subscription.

Analyze and track your performance with our Testing and Analytics Package.